As it is described in the passage, the transnational approach employed by African American historians working in the ...

Zaviea-Gaynor on January 14, 2019

Question #3

I don't understand how the advertising editor's argument fails.

Replies
Create a free account to read and take part in forum discussions.

Already have an account? log in

jgoldman on September 22, 2019

Hey LSAT Max, I have the same question. Please explain.

Victoria on September 22, 2019

Hi @Zaviea-Gaynor and @jgoldman,

Let's start by going through the stimulus.

The magazine editor argues that giving a favourable mention to the products of some regular advertisers in their articles would actually be against the interests of the advertisers. Why? Because for the magazine to effectively advertise these products, they must have loyal readership and in order to have loyal readership, their readers must not believe that their "editorial integrity has been compromised by pandering to advertisers."

In response, the advertising sales director argues that the magazine editor underestimates how sophisticated their readers are. Why? Because the readers recognize that the advertisements are not articles and, therefore, their response to said advertisements does not depend on their opinion of the magazine's editorial integrity.

Right away, we can see that this argument is flawed. Why? Because it doesn't actually respond to the magazine editor's argument.

The advertising sales director says that the reader's response to advertisements does not depend on their opinion of the magazine's editorial integrity. However, the magazine editor's argument is not focused on the readers' response to advertisements; rather, it is focused on whether people will see the advertisements at all. The magazine editor claims that effective advertising requires loyal readership (i.e. people regularly buying the magazine) which, in turn, requires that the readers believe in the magazine's editorial integrity.

In this way, the advertising sales director's argument fails as a response because the magazine editor's argument does not focus on readers' responses to advertisements in the magazine. This is directly restated by answer choice D, making it the correct answer.

Hope this helps! Please let us know if you have any further questions.

@ikeikanwie on July 15, 2020

I struggled with Question 2. I guess I'm confused because I thought the answer was C. I thought, "Well, who is to say that the 1,000 readers surveyed constitutes a majority of the magazine's readership?" In other words, what if 1984 places second only in this 1,000 person sample, but then when say the full magazine readership of 5,000 was surveyed, a different book beat both The Bible and 1984 putting i1984 in 3rd place/making it less influential.

Victoria on May 23, 2021

Hi @@ikeikanwie,

Happy to help!

First, the stimulus does not say that 1984 has influenced the majority of the newspaper's readers; rather, it says "a great number." This does not necessarily mean most, it just means many.

I can see why answer choice (C) is tempting, but answer choice (B) is stronger because it offers us the ability to both strengthen and weaken the argument depending on which 'extreme' we select (i.e. 1 or 499).

Answer choice (C) only offers us the ability to weaken the argument. If there are only 1,000 readers of the newspaper, then the argument is not strengthened because we still do not know how many people chose books other than 1984. All we know is that 1984 was the second most popular selection.

Hope this helps! Please let us know if you have any further questions.